Finding Orthogonal Trajectories (differential equations)

Click For Summary
The discussion focuses on finding orthogonal trajectories for the equation ##\frac{x^2}{a}-\frac{y^2}{a-1}=1## by substituting new variables, where ##x^2=w## and ##y^2=z##. The user is confused about determining the derivatives ##w'## and ##z'## after the substitution. It is clarified that since ##w## is the new independent variable, the derivative should be taken with respect to ##w##, leading to the equation ##(a-1)-az'=0##. The correct interpretation is that ##z'## represents the derivative of ##z## with respect to ##w##. Understanding these derivatives is crucial for solving the problem effectively.
Westlife
Messages
4
Reaction score
2

Homework Statement


Find Orthogonal Trajectories of ##\frac{x^2}{a}-\frac{y^2}{a-1}=1##
Hint
Substitute a new independent variable w
##x^2=w##
and an new dependent variable z
##y^2=z##

Homework Equations

The Attempt at a Solution


substituting ##x## and ##y## I get
$$\frac{w}{a}-\frac{z}{a-1}=1\quad /a(a-1)$$
$$w(a-1)-za=a(a-1)\quad /'$$
$$w'(a-1)-z'a=0$$

Here I wanted to figure out what ##w'##and ##z'## are and here is where I get confused. How do I get them?
since w is now the new independent variable is ##w'= \frac{dw}{dw}=1## and ##z'=\frac{dz}{dw}## or does mean mean something else ?
 
Physics news on Phys.org
Westlife said:

Homework Statement


Find Orthogonal Trajectories of ##\frac{x^2}{a}-\frac{y^2}{a-1}=1##
Hint
Substitute a new independent variable w
##x^2=w##
and an new dependent variable z
##y^2=z##

Homework Equations

The Attempt at a Solution


substituting ##x## and ##y## I get $$\frac{w}{a}-\frac{z}{a-1}=1\quad /a(a-1)$$ $$w(a-1)-za=a(a-1)\quad /'$$ $$w'(a-1)-z'a=0$$

Here I wanted to figure out what ##w'##and ##z'## are and here is where I get confused. How do I get them?
since w is now the new independent variable is ##w'= \frac{dw}{dw}=1## and ##z'=\frac{dz}{dw}## or does mean mean something else ?
Hello @Westlife .
:welcome:
Since the new independent variable is ##\ w\,,\ ## it follows that you take the derivative with respect to ##\ w\,.\ ## That is to say, take ##\ \frac{d}{dw}\,.##

So, yes, you get
##\displaystyle \ (a-1)-a\,z'=0 \text{ , where } z'=\frac{dz}{dw} \,.##​
 
Question: A clock's minute hand has length 4 and its hour hand has length 3. What is the distance between the tips at the moment when it is increasing most rapidly?(Putnam Exam Question) Answer: Making assumption that both the hands moves at constant angular velocities, the answer is ## \sqrt{7} .## But don't you think this assumption is somewhat doubtful and wrong?

Similar threads

  • · Replies 21 ·
Replies
21
Views
2K
  • · Replies 1 ·
Replies
1
Views
1K
  • · Replies 7 ·
Replies
7
Views
1K
  • · Replies 2 ·
Replies
2
Views
1K
  • · Replies 18 ·
Replies
18
Views
3K
Replies
2
Views
1K
  • · Replies 4 ·
Replies
4
Views
906
  • · Replies 8 ·
Replies
8
Views
2K
  • · Replies 5 ·
Replies
5
Views
2K
  • · Replies 7 ·
Replies
7
Views
2K